b91302310
Thanks Received: 13
Atticus Finch
Atticus Finch
 
Posts: 153
Joined: August 30th, 2010
 
 
 

Q20 - Some people have been promoting

by b91302310 Mon Sep 27, 2010 12:27 pm

I know why the correct answer is (C), since it weakens the argument, but not an assumption of the argument. However, for (E), why it could be the assumption of the argument?

Could anyone explain it ?

Thanks!
User avatar
 
bbirdwell
Thanks Received: 864
Atticus Finch
Atticus Finch
 
Posts: 803
Joined: April 16th, 2009
 
 
 

Re: Q20 - Some people have been promoting

by bbirdwell Tue Sep 28, 2010 4:00 pm

Perhaps it's best understood by negating it. If you negate (E), you get: There ARE effective remedies available that people prefer to the mixture.

In this negated form, (E) weakens the conclusion by providing an alternative explanation -- people aren't using the mixture because they prefer something else, not because it doesn't work.

Thus, it must be a necessary assumption. Note that this a common form. The argument says A causes B. One assumption is that it's not the case that C is causing B.
I host free online workshop/Q&A sessions called Zen and the Art of LSAT. You can find upcoming dates here: http://www.manhattanlsat.com/zen-and-the-art.cfm
 
carly.applebaum
Thanks Received: 0
Jackie Chiles
Jackie Chiles
 
Posts: 29
Joined: April 05th, 2012
 
 
 

Re: Q20 - Some people have been promoting

by carly.applebaum Mon May 28, 2012 10:43 am

Can you please explain further why the other answer choices and incorrect? Even with negating them I'm still having a hard time. Especially with proving that C is the correct answer.

Thank you.
 
etwcho
Thanks Received: 12
Forum Guests
 
Posts: 27
Joined: February 24th, 2013
 
This post thanked 1 time.
 
 

Re: Q20 - Some people have been promoting

by etwcho Sun Mar 31, 2013 10:14 am

Although the last post was from a year ago, I will try to explain for others and myself.

All the wrong answers defended the argument by bolstering the conclusion that it was due to the ineffectiveness of the mixture, not some alternate causes/events/effects that prevented people from using it.

a) Bolster's the authors argument by stating there are enough supply of mixture for everyone. What if the mixture was actually effective, but the reason for everyone not using was because the demand far exceeded supply?
b) a medicine can both be effective and still have side-effects. Such as penicillin, it's one of the most widely used antibiotic agent in the world, yet its side effects are deadly to many
c) Argument is about effectiveness of a cold remedy, talking about the prevention of cold is OOS. Thus, correct answer.
d) Similar to a) that it bolsters the demand vs supply. Even with plenty of supply, no one would be using it if no one knew about the product.
e) Again, strengthens by defending the argument. Maybe there is a better tasting synthetic mixture with proven and similar effectiveness to that of the herbal mixture that the people prefer.
User avatar
 
WaltGrace1983
Thanks Received: 207
Atticus Finch
Atticus Finch
 
Posts: 837
Joined: March 30th, 2013
 
This post thanked 1 time.
 
trophy
Most Thanked
trophy
Most Thankful
trophy
First Responder
 

Re: Q20 - Some people have been promoting

by WaltGrace1983 Fri Feb 21, 2014 4:40 pm

I think this question can be very easily approached with the correct mindset. The mindset that you should have when attacking the problem goes something like this...

There are many people who have colds but do not use this mixture
→
Mixture is "obviously not effective."

(1) Think about possible comebacks to the argument. Maybe the mixture is too expensive, tastes bad, has terrible side effects, isn't available, isn't known, etc.

(2) Look for answer choices that rule out these comebacks (similar to what you would be doing with a strengthen question).

(3) Find the answer that doesn't rule out any possible comeback and either weakens the argument, is irrelevant to the argument, or even sometimes is just the last man standing.

Here is what these answer choices are doing:

(A) Rules out the possibility that there isn't enough of the mixture. This is saying that there is enough of the mixture and, according to the reasoning the argument, "most people with colds" should be using it

(B) Rules out the possibility that people aren't using the mixture because its side effects are lousy. This is saying that there definitely aren't many people that are so overcome by the side effects that they choose not to use the mixture

(D) Rules out the possibility that the mixture is relatively unknown. This is saying that people know about it and, according to the reasoning of the argument, this would entail the most people would be using it

(E) Rules out the possibility that this mixture isn't preferred by many people. This is saying that, in general, people don't have strong allegiances towards other remedies

(C) is the only one that really doesn't need to be assumed in order for the argument to work. We don't need to assume anything about the mixture's power, just that the mixture isn't avoided for other reasons besides effectiveness.
 
aaronwfrank
Thanks Received: 2
Vinny Gambini
Vinny Gambini
 
Posts: 23
Joined: August 24th, 2016
 
 
 

Re: Q20 - Some people have been promoting

by aaronwfrank Sat Oct 01, 2016 1:33 pm

(E) Rules out the possibility that this mixture isn't preferred by many people. This is saying that, in general, people don't have strong allegiances towards other remedies

(C) is the only one that really doesn't need to be assumed in order for the argument to work. We don't need to assume anything about the mixture's power, just that the mixture isn't avoided for other reasons besides effectiveness.


Is E wrong because it wrecks the sub-conclusion, which makes the rest of the argument null?

I'm taking "Since most people with colds wish to recover quickly, it follows that almost everyone with a cold would be using it." as the sub.
 
AlexK795
Thanks Received: 0
Vinny Gambini
Vinny Gambini
 
Posts: 4
Joined: March 08th, 2021
 
 
 

Re: Q20 - Some people have been promoting

by AlexK795 Sat Mar 20, 2021 2:52 pm

Continuing on here - C was sorta one of those last man standing choices in that I was able to eliminate the other 4 on my second read. When negating C, I think it actually may strengthen the plausibility of the connection between people not taking the medicine due to its ineffectiveness.

So, C in its originally presented form sorta just weakens the argument straight up - if the mixture is powerful enough to prevent almost everyone who uses it from contracting any further colds, then that would seem to weaken the idea that the medicine is ineffective, and thus that being the reason people don't use it. That's the last thing you'd want an assumption necessary to the argument to do.

I wasn't able to detect this in my timed attempt of the problem, but I believe this is why C is wrong - in real time, however, it was easier for me to just eliminate the other 4 with definitive confidence then select the one that had to be correct.